[Math] A “better” rational approximation of pi

approximation-theoryirrational-numbersnt.number-theory

$355/113$ is a good fractional approximation of $\pi$, because we use six digits to produce seven correct digits of $\pi$.
$$\frac{355}{113} = 3.1415929\ldots$$

Let $R$ be the ratio of the number of accurate digits produced to the number of digits used in the numerator and denominator, then
$$R\left(\frac{355}{113}\right) = \frac 7{3+3} = 1.166666\ldots\,{}$$
Can anyone find a "better" fraction such that $R > 1.16666\ldots\,{}$.

Added: Probably, a similar question would also make sense over a base other than $10$.

Best Answer

Using the idea of the other answer in a different way, if $u$ is the irrationality measure of $\pi$, then except for finitely many $p/q$, we have

$$ \left| \pi - \frac{p}{q} \right| > \frac{1}{q^u} $$

and consequently

$$ \frac{ -\log |\pi - (p/q)| }{\log p + \log q } < \frac{u}{2}$$

and there will be infinitely many fractions $p/q$ that come arbitrarily close to this bound. (and, of course, those finitely many exceptions which may exist that are allowed to exceed it) (and, whatever tiny excesses might arise due to the rounding error in the analysis)

If the irrationality measure if $\pi$ is greater than $2.34$, then there will be infinitely many fractions with a better value of $R$ than the one you found. (although that is not reason to expect any of them are small enough for us to actually find)

If $u < 2.3$, then there can only be finitely many fractions with a better value of $R$. But I have no idea how you would go about checking if any exist at all.

Almost all irrational numbers have irrationality measure $2$; for $\pi$ it's known that $u \leq 7.6063$

Related Question